Answer these questions about the right triangle.

What is the area of the square of the leg 6?

What is the area of the square of the leg 8?

What is the area of the hypotenuse square?

What is the length of the hypotenuse?

Answers

Answer 1

9514 1404 393

Answer:

366410010

Step-by-step explanation:

The area of a square is the square of the side length. Then the side length is the square root of the area.

6² = 36 . . . area of the square of leg 68² = 64 . . . area of the square of leg 836+64 = 100 . . . sum of the two leg squares = area of hypotenuse square√100 = 10 . . . hypotenuse length
Answer These Questions About The Right Triangle.What Is The Area Of The Square Of The Leg 6?What Is The

Related Questions

Một đề thi trắc nghiệm có 10 câu, mỗi câu có 4 phương án trả lời và chỉ có một đáp án đúng. Một sinh viên trả lời một cách ngẫu nhiên, xác suất để sinh viên được 5 điểm là

Answers

Câu trả lời:

0,0584

Giải thích từng bước:

Câu hỏi đưa ra đáp ứng điều kiện cần thiết cho phân phối xác suất nhị thức:

Số câu hỏi, số lần thử, n = 10

Xác suất, p = 1 / số lựa chọn = 1/4 = 0,25

q = 1 - p = 1 - 0,25 = 0,75

Tính xác suất để sinh viên đó được 5 điểm, x = 5;

Gợi lại:

P (x = x) = nCx * p ^ x * q ^ (n-x)

P (x = 5) = 10C5 * 0,25 ^ 5 * 0,75 ^ 5

P (x = 5) = 252 * 0,25 ^ 5 * 0,75 ^ 5

P (x = 5) = 0,0584

C
What is the equation of the line that is parallel to the line y - 1 = 4(x + 3) and passes through the point (4, 32)?
SER
O y=-*x+33
y=-*x+36
O y =
O y = 4x - 16
O y = 4x + 16
Mark this and retum
Save and Exit
Nex
Submit

Answers

9514 1404 393

Answer:

  y = 4x + 16

Step-by-step explanation:

You can replace the point coordinates in the given point-slope form to get ...

  y - 32 = 4(x -4)

Then rearrange to put this into slope-intercept form.

  y = 4x -16 +32 . . . . eliminate parentheses, add 32

  y = 4x +16 . . . . . . . . simplify

A medical device company knows that the percentage of patients experiencing injection-site reactions with the current needle is 11%.

What is the probability that an injection-site reaction occurs for the first time on the 6th patient of the day?

0.0001
0.0614
0.3685
0.4970

Answers

Answer:

0.4970

Step-by-step explanation:

I might be wrong

The probability that an injection-site reaction occurs for the first time on the 6th patient of the day will be 0.4970. Then the correct option is D.

What is probability?

Its fundamental concept is that someone will nearly surely occur. The proportion of positive events in comparison to the total of occurrences.

Then the probability is given as,

P = (Favorable event) / (Total event)

A medical device company knows that the percentage of patients experiencing injection-site reactions with the current needle is 11%.

The probability that an injection-site reaction occurs for the first time on the 6th patient of the day is given as,

P = (1 - 0.11)⁶

P = (0.89)⁶

P = 0.4970

The probability that an injection-site reaction occurs for the first time on the 6th patient of the day will be 0.4970. Then the correct option is D.

More about the probability link is given below.

https://brainly.com/question/795909

#SPJ2

The length of a rectangle is four times its width.
If the area of the rectangle is 100 yd”, find its perimeter.

Answers

Answer: 50yd

Step-by-step explanation:

We know that the area of any rectangle is length times width.  The perimeter is the sum of twice the length and twice the width.

Let width = x

Let length = 4x

Area = 100m2

Next, we can write an equation using these variables and formula for area.

4x2 = 100

x2 = 25

x = -5      and        x = 5

Since the dimensions cannot be negative, we accept the positive value:

x = 5

Next, we can substitute this value of x into the variables.

width = 5 m

length = 20 m

Finally, we can find the perimeter by plugging in these dimensions into the perimeter formula,

Perimeter = 2(5 m) + 2(20 m)

              = 10 m + 40 m

              = 50 m  

If f(x) = x -2 and g(x) = 2x – 6, then g(4)/f(3) =​

Answers

Answer:

Step-by-step explanation:

(2×4-6)/(3-2)=2

Answer:

[tex]{ \tt{f(x) = x - 2}} \\ { \bf{f(3) = 3 - 2 = 1}} \\ \\ { \tt{g(x) = 2x - 6}} \\ { \bf{g(4) = 2(4) - 6 = 2}} \\ \\ { \boxed{ \tt{ \frac{g(4)}{f(3)} = \frac{2}{1} = 2}}}[/tex]

Can anyone help me with this question

Answers

Answer:

50%

Step-by-step explanation:

If angle f is 50 degrees and cuts it to 75 degrees

Find the difference between the two numbers

75-50 = 25

Divide by the correct value

25/50 = 1/2

Change to percent form

50%

Look at the illustration.
What is WX?

Answers

Answer:

O  0.5 units

Step-by-step explanation:

so the first thing we have to do is to calculate for the dilation factor. Taking point G as the reference point, we can see that the distance of point G from rectangle W'X'Y'Z is 1.5 while the distance from rectangle WXYZ is (1.5 + 7.5) = 1.5 / 9 = 1/6

Since WX has an initial measure of 3 units, therefore the measure of W'X' is:

W'X' = 3 units *(1/6) = 0.5 units

A cyclist travels at an average speed of 18 mph for 2 hours.
How far did she travel in miles?

Answers

Answer:

36

Step-by-step explanation:

18 miles per hour so for 2 hours which is double, she should have traveled 36 miles.

Good Luck!

If 21% of kindergarten children are afraid of monsters, how many out of
each 100 are afraid?

Answers

Answer:

The appropriate answer is "21".

Step-by-step explanation:

Given:

Afraid percentage,

p = 21%

or,

  = 0.21

Sample size,

n = 100

As we know,

⇒ [tex]X=np[/tex]

By putting the values, we get

        [tex]=0.21\times 100[/tex]

        [tex]=21[/tex]

Jack is 2 years older than his sister jill. The sum of their ages is 24 years. If jack is x years, find x

Answers

Answer:

x = 13

Jack is 13 years old and Jill is 11 years old.

====================================================

Explanation:

x = Jack's age

x-2 = Jill's age, since she is 2 years younger than Jack

Add up the ages and set the sum equal to 24 to solve for x

x+(x-2) = 24

2x-2 = 24

2x = 24+2

2x = 26

x = 26/2

x = 13

Jack is 13 years old and his sister is x-2 = 13-2 = 11 years old

Check: 13+11 = 24, so the answer is confirmed.

Add the following fractions. See the image below

Answers

Answer:

[tex]\dfrac{19}{840}[/tex]

Step-by-step explanation:

The given fraction is:

[tex]\dfrac{1}{168}+\dfrac{3}{180}[/tex]

We need to solve it.

The LCM of 168 and 180 is 2520.

So,

[tex]\dfrac{1}{168}+\dfrac{3}{180}=\dfrac{15+3\cdot14}{2520}\\\\=\dfrac{19}{840}[/tex]

So, the required answer is equal to [tex]\dfrac{19}{840}[/tex].

20 POINTS MATH PROBLEM

Answers

Answer:

D. x=36

Step-by-step explanation:

3x-5=103

3x=103+5

3x=108

x=36

The upwards acceleration of a small rocket at time t s is given by a = 16 − 1.5t. The rocket is subject to this
acceleration for 3 seconds. Given that it starts from rest at t = 0, calculate the height reached by the rocket in
this time

Answers

Answer:

11.5

Step-by-step explanation:

plug 3 in for x

then solve

Sketch the region enclosed by the given curves and calculate its area.
y=4-x^2 ,y=0

The answer is 32/3. But how do I get to that answer?

Answers

Answer:

Step-by-step explanation:

1.) we need to find the bounds of integration which is just the points of intersection

here is it (-2,0) and (2,0)

which means we will integrate from -2 to 2

next, we take the upper equation and subtract that from the lower one

kind of confusing but it would look like (sketch it out if you're not sure)

(4-x²)-0= 4-x²

then we can integrate

[tex]\int\limits^2_{-2} {4-x^2} \, dx =4x-\frac{x^3}{3}|_{-2}^{2}=(4*(2)-\frac{2^3}{3})-(4(-2)-\frac{-2^3}{3})=5.333333-(-5.3333333)= 10.666666667=\frac{32}{3}[/tex]

To estimate the difference we need four averages for the categorized groups i.e., control group before change, control group after change, treatment group before change and treatment group after change.

a. True
b. False

Answers

Answer:

b. False

Step-by-step explanation:

In a research study, when a researcher wants to find the impact of a new treatment, then the researcher randomly divides the the study participants into two groups. The groups are :

-- control group

-- treatment group

The control group is a group that is used to establish the cause-and-effect relationship by making the effect of an independent variable isolate. It receives no treatment or some standard treatment for the which the effect is already known.

The treatment group receives the treatment for which the effect the researcher is interested in.

Thus the averages of the four categorized groups are not required for estimating the difference.

Therefore, the answer is FALSE.

What is the value of z2 if 3z1 = 6 + 18i and z1 + z2 = 5 + 9i?

Answers

Answer:

z2=-3-3i

Step-by-step explanation:

3z1+0z2=6+18i

z1+z2=5+9i (×3)

3z1+3z2=15+27i

3z1+0z2=6+18i (-) =

-3z2=9+9i

z2=-3÷(9+9i)

z2=-3-3i

Determine which value best approximates the length of the arc represented by the integral ∫_0^1 √1 + [d/dx(4/x+1)]² dx.
(Make your selection on the basis of a sketch of the arc and not
by performing any calculations.)
(a) 10
(b) -5
(c) 2
(d) 4
(e) 1

Answers

Answer:

Option C

Step-by-step explanation:

From the question we are told that:

Length of arc integral

[tex]l=\int_0^1 \sqrt{1 + [\frac{d}{dx}(\frac{4}{x+1})]^2 dx}[/tex]

The Sketch is attached below

From the Graph

Approximation gives length of arc as

[tex]l=\sqrt{5}[/tex]

[tex]l=2[/tex]

Option C

The football and soccer teams at Juan's middle school are selling movie tickets. He wants to determine the average
number of tickets sold at the school. First, he surveyed eight random players on the football team. Then, he surveyed
every third athlete on the football and soccer team. Which sampling technique should produce a more representative
sample?
The first sampling method, surveying eight random football players, is the most representative.
O Neither sample will be representative.
Both samples should be exactly the same.
The second sampling method, survey.ing the both teams, is the most representative.

Answers

Step-by-step explanation:

both samples should be exactly the same

Simplify log2 20-log2 30.​

Answers

Answer:

[tex] log_{2}(20) - log_{2}(30) \\ log_{2}(20 \div 30) \\ log_{2}(0.667) [/tex]

hope this helps you

Brainliest appreciated

log1(20)-log2(30)

log2 (20÷30)

log2=0.667

help pleaseeee it’s timed!!!

Answers

Answer:

C

Step-by-step explanation:

The solution triangle is attached below :

Tonobtinnthe Angle formed, θ; we apply trigonometry ;

Using ;

Cos θ = Adjacent / hypotenus

Cos θ = 4 / 7

θ = Cos^-1(4/7)

θ = 55.15°

θ = 55°

Four students want to have their picture taken together. They will stand side-by-side for the picture. In how many different ways can the four students be arranged to take a picture?

Show your work, please :')

Answers

Answer:

24 ways

Step-by-step explanation:

This is the permutation of 4:

4P4 = 4! = 1*2*3*4 = 24 ways

[tex]\huge\qquad \mathbb{\fcolorbox{red}{lavenderblush}{✰Answer}}[/tex]

✶⊶⊷⊶⊷❍❁❥❀❥❁❍⊶⊷⊶⊷✶

Four students want to have their picture taken together. They will stand side-by-side for the picture. In how many different ways can the four students be arranged to take a picture

so we have to find the permutation of 4

4×3×2×124

.°. In 24 different ways can the four students be arranged to take a picture

Find the area of the rectangle if the perimeter is 52 cm

Answers

Answer:

No solution is possible from the information provided

Step-by-step explanation:

I'm not good with ordering fractions from smallest to largest. Can help anyone help with this problem?

Answers

5/8,4/6,3/4,1/2 this is the answer without simplifying

I 0 ×I 1× I 2 ×I 3×I 4

Answers

Answer:

the answer is twelve by multiplying them all.

Answer:

2 , 40,420

Step-by-step explanation:

10 × 11 × 12 × 13 × 14

1,320 × 182

2,40,240

What is the slope intercept form of the equation of the line shown below

Answers

Answer:

[tex]y=\frac{4}{3}x-4[/tex]

Step-by-step explanation:

----------------------------------------

The slope-intercept form formula is: [tex]y=mx+b[/tex]

The [tex]m[/tex] stands for the slope and the [tex]b[/tex] stands for the y-intercept.

By looking at the graph, I can figure out that the y-intercept is -4 because y-intercept is where the lines cross the y-axis and in this graph, the line crosses the y-axis at (0,-4).

The slope is [tex]\frac{4}{3}[/tex] because to get to the ordered pair (3,0), from (0,-4), you would have to go up 4 and over 3 to the right so it's [tex]\frac{4}{3}[/tex]

So now, if we insert the values in the formula, it would be [tex]y=\frac{4}{3}x-4[/tex]

----------------------------------------

Hope this is helpful.

Answer:

y = 4/3x -4

Step-by-step explanation:

First find the slope

m = ( y2-y1)/(x2-x1)

    = ( 0 -  -4)/( 3 - 0)

   = (0+4)/( 3-0)

   = 4/3

The y intercept is -4

The slope intercept form of the equation is

y = mx+b  where m is the slope and b is the y intercept

y = 4/3x -4

it cost $246 to turf a lawn of 43m ² . how much will it cost to turf a lawn of 61m square 2 .​

Answers

Answer:

$348.96

Step-by-step explanation:

246÷43=5.72

61-43=18

18×5.72=102.96

102.96+246=348.96

Find the value of PR if Q is between P and R
when PQ=25, PQ=2x+1, and QR=x.
X
ат
X=8

Answers

Answer:

[tex]PR = 37[/tex]

Step-by-step explanation:

Given

[tex]PQ = 25[/tex]

[tex]PQ = 2x +1[/tex]

[tex]QR = x[/tex]

Required

PQ

Since Q is between the given points, then:

[tex]PR = PQ + QR[/tex]

This gives:

[tex]PR = 2x + 1 + x[/tex]

Collect like terms

[tex]PR = 2x + x+ 1[/tex]

[tex]PR = 3x + 1[/tex]

Next, solve for x

We have:

[tex]PQ = 25[/tex]

[tex]PQ = 2x +1[/tex]

This gives:

[tex]2x + 1 = 25[/tex]

Collect like terms

[tex]2x = 25 -1[/tex]

[tex]2x = 24[/tex]

Divide by 2

[tex]x = 12[/tex]

So:

[tex]PR = 3x + 1[/tex]

[tex]PR = 3 * 12 + 1[/tex]

[tex]PR = 37[/tex]

After a new product is launched the cumulative sales S(t) (in $1000) t weeks after launch is given by:

S(t) = 72/1 + 9e^-0.36t

Required:
a. Determine the cumulative amount in sales 3 weeks after launch.
b. Determine the amount of time required for the cumulative sales to reach $70,000.
c. What is the limiting value in sales?

Answers

Answer:

$17.750 ; 15.979 ; 72

Step-by-step explanation:

Given that :

Cummulative sales, S(t) is represented by the equation :

S(t) = 72/(1 + 9e^-0.36t)

Cummulative sales after 3 weeks :

Put t = 3 in the equation, as t = time after launch

S(3) = 72/(1 + 9e^-0.36(3))

S(3) = 72 / (1 + 9e^-1.08)

S(3) = 72 / (1 +3.0563597)

S(3) = 72 / 4.0563597

S(3) = 17.749905 = $17.750 thousands

Amount of time required for sales to reach 70000

S(t) = 72/(1 + 9e^-0.36t)

S(t) = 70

70 = 72/(1 + 9e^-0.36t)

70 * (1 + 9e^-0.36t) = 72

(1 + 9e^-0.36t) = 72 / 70

1 + 9e^-0.36t = 1.0285714

9e^-0.36t = 1.0285714 - 1

9e^-0.36t = 0.0285714

e^-0.36t = 0.0285714 / 9

e^-0.36t = 0.0031746

Take the In of both sides ;

In(e^-0.36t) = In(0.0031746)

-0.36t = - 5.752573

t = - 5.752573 / - 0.36

t = 15.979

About 16 weeks

The limiting value in sales :

Take the limit as t - - > ∞

S(t - - > ∞) = 72/(1 + 9e^-0.36t)

Put t = 0

S(0) - - > 72 / (1 + 0)

72 / 1

= 72

Suppose that g(x)= f(x)+ 6. Which statement best compares the graph of g(x) with the graph of f(x)?

A. The graph of g(x) is the graph of f(x) shifted 6 units down.

B. The graph of g(x) is the graph of f(x) shifted to the right.

C. The graph of g(x) is the graph of f(x) shifted 6 units to the left.

D. The graph of g(x) is the graph of f(x) shifted 6 units up.

Answers

Answer:

D

Step-by-step explanation:

The + 6 moves it up 6 units.

The correct answer is (D) "The graph of g(x) is the graph of f(x) shifted 6 units up."

What is the function?

A relationship between a group of inputs and one output is referred to as a function. In plain English, a function is an association between inputs in which each input is connected to precisely one output. A domain, codomain, or range exists for every function. Typically, f(x), where x is the input, is used to represent a function.

When we add a constant to a function, such as in the case of g(x) = f(x) + 6, it will shift the graph of f(x) upward by 6 units.

This is because, for any value of x, the value of f(x) will be added to 6, resulting in a vertical shift of the entire graph.

Option (A) is incorrect because adding 6 to f(x) would shift the graph up, not down.

Option (B) is incorrect because adding a constant to a function does not cause it to shift horizontally.

Option (C) is incorrect because adding 6 to f(x) would shift the graph right, not left.

D. The graph of g(x) is the graph of f(x) shifted 6 units up. Adding a constant term to a function will shift the graph of the function vertically. In this case, adding 6 to f(x) will shift the graph of f(x) upward by 6 units, resulting in the graph of g(x).

Learn more about function here:

https://brainly.com/question/29633660

#SPJ7

Please help me with this question

Answers

C. None from the above
Other Questions
A company reports the following income statement and balance sheet information for the current year: Net income $216,060 Interest expense 38,130 Average total assets 2,290,000 Determine the return on total assets. If required, round the answer to one decimal place. 7.2962.4 full answer please Zumwalt Corporation's Class S bonds have a 12-year maturity, $1,000 par value, and a 5.75% coupon paid semiannually (2.875% each 6 months), and those bonds sell at their par value. Zumwalt's Class A bonds have the same risk, maturity, and par value, but the A bonds pay a 5.75% annual coupon. Neither bond is callable. At what price should the annual payment bond sell? (hint: Bonds with same risk should have same effective rate of return, or YTM). Maxim Corp. has provided the following information about one of its products:Date Transaction Number of Units Cost per Unit1/1 Beginning Inventory 285 $ 157 6/5 Purchase 485 $ 177 11/10 Purchase 185 $ 217 During the year, Maxim sold 570 units.What is cost of goods sold using the average cost method? (Do not round intermediate computations.) how does fertilization of angiosperm differ from fertilization in bryophytes digestion that takes place in the mouth. help plzzz I don't understand this question Based on your reading in Matthew Chapter 10, Jesus sent each of the twelve apostles forth as " ______ .sheep in the midst of wolvesmasters over slavesa servant like his mastera disciple like his teacher For EASY BRAINLIEST!!! Answer that question My future plans to be an Airline Pilot. (50words) paragraph.As a student i need for this question answer. 26._____The first trophic level is usually occupied by herbivores. 27.__True___It is possible for a plant to be a carnivore. 28._____A habitat refers to the role of a species in its ecosystem. 29._____Biotic factors include sunlight, soil, temperature, and water. 30._____A food web can be represented by many interconnected food chains. 31._____There are rarely more than four trophic levels in an ecosystem. 32._____Like water, energy is also recycled through an ecosystem. HELLPPP DUE IN 10 MINUTES Assignment1Which is rational?a. 64b. 17C. 67.714819...22. Which is irrational?a. 75.106c. 101Identify each of the following as Rational or Irrational.3 -424 65.4279598166257 57.013013..818.44333777...9-139.042410 Mga taong nananakot o nanggulo sa pamahalaan at bansa Which statement best describes how this excerpt contrasts with the main idea in "Do not go gentle into that good night"? Please help!! The slope of a line that passes through the points (-6, w) and (-10, 4) ls 1/8. What is thevalue of w? Jessie and Paul have worked in the same office at DEF Insurance LLC for 6 years. Jessie has always taken extra care to follow the office norms and ensure that everyone has a happy and harmonious working experience. Paul has been very driven during his years at DEF Insurance, and though he started as an insurance agent, he has since been promoted to the agency manager and proudly displays all of his awards on the wall of his new office. According to four drive theory, Paul most likely has a high: a. drive to acquire b. drive to defend c. drive to comprehend d. drive to achieve What is the product on this Brainly test Put these numbers in order from greatest to least.0.32,1/2,0.66, and 0.45 how many joules of work are done against a cart when a force of 50N pushes it 1000m anway